A figure is shown
9 in.
3 in.
2 in.
3 in.
Which expression shows how to find the area of the figure?
A. 9x2 + 3x3
O C. 9+3+2+3
B. 9x 3 + 3 x 2
OD. 9 X5
What is the area of the figure? Enter the answer in the box.

A Figure Is Shown9 In.3 In.2 In.3 In.Which Expression Shows How To Find The Area Of The Figure?A. 9x2

Answers

Answer 1

Answer:

C. 9×3 + 3×2

The area of the figure is 33 in.

Answer 2

Step-by-step explanation:

Area is:9x5

Hope i could help u

A Figure Is Shown9 In.3 In.2 In.3 In.Which Expression Shows How To Find The Area Of The Figure?A. 9x2

Related Questions

Juanita had 20 dollars to spend on 3 gifts. She spent 8 3 4 dollars on gift A and 4 1 2 dollars on gift B. How much money did she have left for gift C?

Answers

Answer:

the amount left for gift c is $7.54

Step-by-step explanation:

The computation of the amount left for gift c is shown below:

= Total money to spend - spent on gift A - spent on gift B

= $20 - $8.34 - $4.12

= $20 - $12.46

= $7.54

Hence, the amount left for gift c is $7.54

7.5 is what percent of 50?

Answers

Answer:

15%

Step-by-step explanation:

HELP I NEED HELP ASAP

A population of mice grows at a rate of 7% each year. Which function, f(t), could represent the total number of mice after t years?

Answers

Answer:

B

Step-by-step explanation:

As it is mentioned in the question that the mice grows at a rate of 7%, it would be answer B. Hope this helps, thank you !!

plz i need help hurry
and how do you mark brainliest

Answers

Answer:7 times 3.14=21.98

Step-by-step explanation:click the crown at the bottom of my answer

Answer:

43.98

Step-by-step explanation:

arc length =2* pie * r

Solve the next equation (5 x - 3)³ = 8​

Answers

Answer:

x=1

Step-by-step explanation:

(5x-3)3=8

taking the cube of both sides we get

5x-3=2

adding three to both sides we get

5x=5

dividing by five on both sides we get

x=1

if you found my answer helpful please mark as brainliest.



What are the coordinates of the vertex?

Answers

0,-8 are the coordinates of the vertex.
please help me this is my test for the review of all the math i learned

When the angle of elevation to the sun is 42 a flagpole casts a shadow that is 21 ft long. What is the height of the flagpole to the nearest foot?

Answers

Answer:

18.91 cm

Step-by-step explanation:

Let x represent the height of the flag pole

Tan= opposite/ adjacent

Tan 42°= x/21

x= Tan 42° × 21

= 0.9004 × 21

= 18.91 cm

Hence the height is 18.91 cm

The following box plot shows points awarded to dance teams that competed at a recent competition:

Which dot plot best represents the box plot data?

(answer choices are in order btw) Answer choices:

Answers

Answer:

The following box plot shows points awarded to dance teams that

Step-by-step explanation:

Which dot plot best represents the box plot data?

(answer choices are in order btw) Answer choices:

What is the solution to the system of equations below?
y = 5x+6 and Y--x-4

Answers

Answer:

the answer is number ten!!!!!!!!!!!!

Step-by-step explanation:

V (x) = (18 - 2x) (24-2x) (x)
what is a reasonable domain for V in this context? ​

Answers

Answer:

What are you asking?

Step-by-step explanation:

The domain of the given expression is all real numbers.

What is domain and range of the function?

The domain and range are defined for a relation and they are the sets of all the x-coordinates and all the y-coordinates of ordered pairs respectively. For example, if the relation is, R = {(1, 2), (2, 2), (3, 3), (4, 3)}, then:

Domain = the set of all x-coordinates = {1, 2, 3, 4}

Range = the set of all y-coordinates = {2, 3}

The given function is v(x) =(18-2x)(24-2x)(x).

The domain of the expression is all real numbers except where the expression is undefined. In this case, there is no real number that makes the expression undefined.

Interval Notation: (-∞, ∞)

Set-Builder Notation: {x|x∈R}

Therefore, the domain of the given expression is all real numbers.

To learn more about the domain and range visit:

brainly.com/question/28135761.

#SPJ2

Do 1/4 and 4/16 form a proportion

Answers

Answer:

There is a difference between FRACTION and PROPORTION.

Explanation of fraction:

On a card board sheet draw a square or rectangle or any simple shape of your choice. Divide your preferred shape into 4 equal parts. One portion or one part of the divided shape is 1/4. Three (3 ) parts of the divided shape equal 3/4. Also if the shape is divided into twenty (20). One (1) part is 1/20, sixteen(16) parts equal 16/20.

Explanation of Proportion:

If an item is shared in a proportion of 3:4,it means that A gets 3/(3+4) of what is to be shared , that is 3/7 of the bulk item to be shared. B gets 4/(3+4) of what is to be shared, that is 4/7 of the bulk item to be shared.

Also if an item is shared in a proportion of 16:20 , A gets 16/(16+20)=16/36 of Bulk item while B gets 20/(16+20)= 20/36 of the Bulk item being shared.

In effect Fractions and Proportions are related but different.

What is the slope of the line in this picture?



Answers

Answer:

The slope is 3/2

HELP PLEASEEEEEE!!!!!!!!!!!!!!! TY

Answers

The answer is 24w - 16u + 48

PLEASE HELP! ASAP please help

Answers

Answer:

( 3 , -5 )

Step-by-step explanation:

5(x+2) = 20

15 points

Answers

Answer:

Step-by-step explanation:

remove the parenthesis

5x+10=20

5x+10-10=20-10

5x=10

x=2

the table below shows the heights of several books. Jean stacks a dictionary on top of her novel. How high is the stack of the two books?

A:between 5 and 5 1/2 inches
B:between 5 1/2 and 6 inches
C:between 4 1/2 and 5 inches

GIVING BRAINLIEST IF CORRECT PLSS HEP QUICKK!!!!!

Answers

Answer:

Between 5 and 5 1/2 inches

Step-by-step explanation:

Between 5 and 5 1/2 inches

Please help me........

Answers

Answer: i don't know

Step-by-step explanation:

hi i  don't know any of this stuff i hope you don't learn anything Bye  

Solve for y : y/2 - 5= -11 (I need help I’m a bit confused).

Answers

y/2 - 5 = -11
multiply each side by 2
y - 10 = -22
add 10 to both sides
y = -12

Which of the following correctly describes the quotient below?

A.
rational

B.
irrational

C.
neither rational nor irrational

D.
both rational and irrational

Answers

Answer:a

Step-by-step explanation:

27. A multiple choice test contains 5 questions, and each question has four possible responses.
How many different answer keys are possible?
A. 20
B. 120
C. 256
D. 1024

Answers

Answer:

D. 1024

Step-by-step explanation:

Each question can be answered 4 ways.

4 * 4 * 4 * 4 * 4 = 1024

D. 1024

A rectangular pyramid has a volume of 2032 cm. It is dilated by a scale factor of 3.
What will be the volume of the new rectangular pyramid?

Answers

Answer:

dunno

Step-by-step explanation:

Answer:

6,096

Step-by-step explanation:

2,032 x 3 = 6,096

the maximum number of students in a classroom is 26. if there are 16 students sign up for the art class,how many more students can join the class without exceeding the maximum? ues ">=" for more than and "<=" for less than.

and thanks for answering​

Answers

Maximum 12 fr longgg

Answer:

x = 16 ≥ 26

Step-by-step explanation:

x = 10

10 + 16 = 26

Sorry for the delay, my man!

For what value of q is the equation -13(q+4)=7q+16 true?

Please help! Thanks!

Answers

Answer:

I love algebra anyways

The ans is in the picture with the  steps how i got it

(hope this helps can i plz have brainlist :D hehe)

Step-by-step explanation:

Simple question: What is the difference between 32% vs 15%?

Answers

Answer:

Look below :)

Step-by-step explanation:

Well one of them is a bigger percentile, so the difference is 32-15. So the answer is 17. I hope this helps!

Which of the following sets of sides does not form a right triangle?
6.8.10
8.15, 17
7.24.26
5, 12, 13?

Answers

Answer:

7, 24, 26

Step-by-step explanation:

6 squared is 36 and 8 squared is 64. Add them together and you get 100, which is 10 squared. Do the same for the other ones and you'll notice that 7 squared and 24 squared added together is 625, which is not 26 squared. In my head 25 squared is 625(check if you don't believe me) so I know that the answer is the third one. The basic formula for a right triangle is a^2 + b^2 = c^2, and that's how you find if three numbers form a right triangle or not. Hope this helps :)

Find the seventh term of the geometric sequence 1, 2, 4, ... and the sum of the first seven terms. t7= S7=

Answers

Answer:

Seventh term is 64

Sum of the first seven terms is 127

Step-by-step explanation:

The common ratio is 2

[tex]a_{n} = a_{1} r^{n - 1}[/tex]    

n = 7 and the first term is 1.  So,

[tex]a_{7} = 1(2^{7 - 1} ) = 2^{6} = 64[/tex]

Seventh term is 64

[tex]S_{n} = \frac{a_{1} (r^{n} -1)}{r - 1}[/tex]

[tex]S_{7} = \frac{1(2^{7} -1)}{2 - 1}[/tex]

    [tex]= \frac{128 - 1}{1} = 127[/tex]

Sum of the first seven terms is 127

1/6 + (-7/9) in simplest form

Answers

Answer:

Step-by-step explanation:

3/18 + (-14/18)= -11/18

How would you write this number in numeric form?

twenty-two and ninety-four thousandths

Answers

Answer:

22.094

Step-by-step explanation:

Ella practices piano for 45 minutes she spends m minutes longer working on her homework than she does practicing the piano

Answers

Answer: See explanation

Step-by-step explanation:

Your question isn't complete but let's assume that you want to know the amount of time that she spends working on her homework.

Since we are informed that Ella practices piano for 45 minutes and that she spends m minutes longer working on her homework than she does practicing the piano, the expression to get the number of minutes that she uses on her homework will be:

= 45 + m

For example let's say that she spends 7 minutes more in her homework. Therefore, the total minutes spent on the homework will be:

= 45 + 7

= 52 minutes

PLZ HELP ME :(
Tanisha lives in an apartment and pays the following expenses each month: electric bill,$42.60; TV streaming service,$27.99; and rent,$587.70. Estimate her total expenses for the month by first rounding each value to the nearest tens place.
A.)$659
B.)$650
C.)$670
D.)$660

Answers

Answer:

D)$660

Step-by-step explanation:

Add all of them up and you get $658.29 the number in the tens place is 5 and the number on the right is 8 so you round up.

Other Questions
calculate how much of an iceberg is beneath the surface of the ocean, given that the density of ice is 917 kg/m3, and salt water has density 1,025 kg/m3. Rousseau begins The Social Contract with the claim that "Man is born free, and everywhere he is inchains. " What are these restrictions on man's liberty? How are they affected by the social contract? The author wants the reader to understand the strength and skills of Davy Crockett. Give 3 details from the story that explain his strength and skills Mobile push notifications are especially effective under what condition?When they are activity sensitiveWhen they are time sensitiveAll of theseWhen they are location sensitive explain why the generator voltage regulation is different for different load power factors. A whale population of 34 is growing at an annual rate of 12%. How many whales will be there in 10 years? Were supposed to use the function y=a(1 +or- r)^t for exponential growth or decay.) TRUE/FALSE. A party who speaks with a reckless disregard for the truth not knowing of the falsity of his or her words cannot be liable for fraud. Find the minimum and maximum values of y=147sec on the interval [0, /3] Gain on disposal of plant assets is included in the other revenues and gains section of the income statemer All of the following items would be reported as other revenues and gains for a merchandiser except O gain on disposal of plant assets. interest revenue rent revenue. sales revenue. what are the formal charges on the central atoms in each of the reducing agents?a. +1b. -2c. -1d. 0 how many bonds and lone pairs are in one molecule of hydrazine, n2h4? What is the output of the following code snippet?fibonacci = {1, 1, 2, 3, 5, 8}primes = {2, 3, 5, 7, 11}both = fibonacci.union(primes)print(both)a. {1, 2, 3, 5, 8} b. {1, 2, 3, 5, 7, 8, 11}c. {2, 3, 5}d. {} if george's mps is 0.75 and he earns an additional $1,000, how much would he save?a. $250 b. $750 c. $1,333 d. $4,000 Repeated sampling of a certain production process reveals the average of all sample means = 40 cm, the average of all sample ranges = 0.5 cm. These values are based on a constant sample size of n = 6. The control limits for an R chart are:A. zero to 2.504B. zero to 1.002C. 37.996 to 42.004D. -1.504 to 2.504E. none of the above draw the major organic product from reaction of 1-butyne with bh3 in thf, then h2o2, oh-. Explain why the alternating p-series: 1 1 2 p 1 3 p 1 4 p converges for every p > 0. for what p-values is it absolutely convergent? conditionally convergent? (a) Use data in Appendix c to estimate the boiling point of benzene, C6H6(l) (b) Use a reference source, such as the CRC Handbook of Chemistry and Physics, to find the experimental boiling point of benzene. How does the concentration of enzymes affects the reaction? Determine whether the systems with the following characteristic equation (CE) is stable by using Routh-Hurwitz criterion. s +45 +35'+25 +s?+4s+4-0 The rate at which an assembly line workers efficiency E (expressed as a percent) changes with respect to time t is given by E'(t)= 50-4t, where t is the number of hours since the workers shift began. Assuming that E(1)=96 find E(t).